Linear Algebra - Bilinear Forms and Change of Basis

Click For Summary
SUMMARY

The discussion focuses on finding the matrix of a bilinear form relative to two different bases, Alpha' and Beta'. The original matrix A, derived from the bases Alpha and Beta, is given as:

3 -4
4 -5
-1 2

To find the matrix of the bilinear form relative to the new bases, users must compute the transition matrices Q and P. The relationship is defined by the equation QTAP, where T denotes the transpose. The challenge lies in determining the correct transition matrices from the original bases to the new bases.

PREREQUISITES
  • Understanding of bilinear forms and their matrix representations
  • Knowledge of basis transformation and transition matrices
  • Familiarity with matrix operations, including transposition and inversion
  • Proficiency in linear algebra concepts, particularly change of basis
NEXT STEPS
  • Learn how to compute transition matrices for basis transformations
  • Study the properties of bilinear forms and their matrix representations
  • Explore the concept of matrix inversion in the context of linear transformations
  • Investigate practical applications of bilinear forms in various fields of mathematics
USEFUL FOR

Students and professionals in mathematics, particularly those studying linear algebra, as well as educators looking to enhance their understanding of bilinear forms and basis transformations.

TorcidaS
Messages
5
Reaction score
0

Homework Statement


Find the matrix of f relative to Alpha' and Beta'.

Alpha' = [(1,0,0), (1,1,0), (2,-1,1)]
Beta' = [(-13,9), (10,-7)]


The question originally reads that f is a bilinear form.

I've found a (correct according to answer key) matrix A that is

3 -4
4 -5
-1 2

from a given basis of
Alpha = [(1,0,0), (1,1,0), (1,1,1)] and
Beta = [(1,-1), (2,-1)]

Homework Equations



If Q is the matrix of transition from Alpha to basis Alpha' of U and P is the matrix of transition from Beta to basis Beta' of V, then QTAP = matrix of f relative to Alpha' and Beta'.
T meaning transpose.

The Attempt at a Solution


I've gotten oh so far with this question, but I'm stuck here in the final part. I'm confused how to attain said matrices Q and P from the relevant equations. If I'm not terrible mistaken, I think I have to do something involving inverses but I'm at a loss here.

Any help will of course be greatly appreciated.
 
Physics news on Phys.org
You want to express the vectors of one basis in terms of the other basis.
 
Question: A clock's minute hand has length 4 and its hour hand has length 3. What is the distance between the tips at the moment when it is increasing most rapidly?(Putnam Exam Question) Answer: Making assumption that both the hands moves at constant angular velocities, the answer is ## \sqrt{7} .## But don't you think this assumption is somewhat doubtful and wrong?

Similar threads

  • · Replies 6 ·
Replies
6
Views
2K
Replies
4
Views
2K
Replies
8
Views
2K
  • · Replies 2 ·
Replies
2
Views
2K
  • · Replies 1 ·
Replies
1
Views
2K
  • · Replies 3 ·
Replies
3
Views
2K
  • · Replies 6 ·
Replies
6
Views
2K
  • · Replies 10 ·
Replies
10
Views
1K
  • · Replies 16 ·
Replies
16
Views
2K
  • · Replies 1 ·
Replies
1
Views
2K